LSAT and Law School Admissions Forum

Get expert LSAT preparation and law school admissions advice from PowerScore Test Preparation.

 Administrator
PowerScore Staff
  • PowerScore Staff
  • Posts: 8917
  • Joined: Feb 02, 2011
|
#23221
Complete Question Explanation

Parallel Reasoning-SN. The correct answer choice is (A)

This Parallel question is a tricky conditional reasoning question. The stimulus give you a conditional reasoning statement as the second sentence (notice the "if") but then tells you that its subject does not fall under that reasoning, so it should not be applied. This abstraction can help you find the answer. The stimulus states that a business that is not profitable should be closed. We know from the first sentence that the county park system is not profitable, but then we find out it is not a business. You might diagram the second sentence as: if B (something is a business) and U (and it's unprofitable) then C (it should be closed): B and U → C. We then find out not B, so the conclusion is not that the county park system should not be closed. We cannot infer that except by Mistaken Negation. We can only infer that being unprofitable does not justify closing it.

Answer choice (A): This is the correct answer choice, as it matches exactly the structure of the stimulus. If a show is in prime time and it fails to attract a large audience, it should be cancelled. However, the documentary does not air during prime time, so this reasoning does not justify (is not sufficient) to cancel it. Notice that the conclusions of the two arguments match exactly. One uses "does not justify" and the other uses "is not sufficient" and we know from Critical Reasoning questions that these two statements are exactly the same.

Answer choice (B): Notice that the conclusion does not match. Here the conclusion is a "should" not a "does not justify" like the stimulus. Also, neither one of the sufficient terms is negated like it is in the stimulus and answer choice A. According to the information we are given, OKESA appears to manufacture and market automobiles in the USA (it does not matter whether it also manufactures bicycles).

Answer choice (C): Again, this conclusion is a "should" not a "does not justify." The writers want you to think that the stimulus says the county parks "should" be closed, but the stimulus does not say that. It simply says that being unprofitable does not alone justify a closure.

Answer choice (D): The conclusion is once again a "should" and the reasoning is completely different. There is no conditional reasoning here; there are no sufficient or necessary clauses.

Answer choice (E): Although this answer choice does contain conditional reasoning (notice the "since" and the "should"), it is really only one premise and has not other information or principles to back it up and it also does not match the conclusion of the stimulus.
 adlindsey
  • Posts: 90
  • Joined: Oct 02, 2016
|
#31509
the conclusion is not that the county park system should not be closed.
So the conclusion is that it should be closed, because of the double negatives?
 Adam Tyson
PowerScore Staff
  • PowerScore Staff
  • Posts: 5153
  • Joined: Apr 14, 2011
|
#31523
Not quite, adlindsey - what our explanation is telling you is that "the park should not be closed" isn't the conclusion of the argument. Yes, that's a double negative (it's not "not"), but in this case our two negatives don't make a positive. Instead, they allow for a neutral alternative - that we just don't know.

The conclusion is that the evidence does not, by itself, justify closing the park. It also doesn't justify keeping it open. We just don't know what to do based on the information we have.

I hope that didn't not clear it up!
 pranavshah7887
  • Posts: 6
  • Joined: Jul 10, 2020
|
#78126
I am sorry, but I am a bit thrown off by the explanation given. Could someone please dilute the explanation a bit more, perhaps making it easier for people like me to understand. Below is the way i approached this question-

1) County business unprofitable -> close them
2) But parks are not business so the above premise doesn't hold in entirety and becomes-
3) Even if county parks unprofitable-> ~ close them [not getting the option to strike it off].

Will the correct representation of answer choice A be as -

1) ~Large Audience->Cancel P.T.V [prime time TV]
2) But documentaries are not aired during prime time, so the above premise doesn't hold in entirety and becomes-
3) ~Large Audience[since it's a small audience]-> ~Cancel documentaries.

Both the reasoning ,now, look similar and is that the reason why A is correct?

Please explain a bit more, and let me know if my reasoning is wrong.

Regards
Pranav
User avatar
 KelseyWoods
PowerScore Staff
  • PowerScore Staff
  • Posts: 1079
  • Joined: Jun 26, 2013
|
#78872
Hi Pranav!

Your reasoning is correct. The basic structure of both the stimulus and answer choice A are here's a conditional rule, but the conditional rule does not apply in this specific case, so the fact that this specific case meets the sufficient condition is not enough to prove the necessary condition.

Hope this helps!

Best,
Kelsey

Get the most out of your LSAT Prep Plus subscription.

Analyze and track your performance with our Testing and Analytics Package.